The question stem says of the following judgments, which one most closely conforms to the principle above? This is a rarer type of question though we have seen it plenty before. They're asking us to take the principle in the stimulus which is a conditional statement and push it into the arguments in the answers to see where it fits. But that’s like a PSA question. Instead of the stimulus containing an argument searching for a conditional in the answer, it's the other way around. The stimulus contains a conditional searching for an argument. This is a cosmetic difference.

The stimulus lays down two jointly sufficient conditions for justified governmental interference with an individual’s actions. The two conditions are:

  1. The action would increase the likelihood of harm to others; and
  2. The action is not motivated by a desire to help others.

If both conditions are met, then the government is justified in interfering with the individual’s action.

As an aside, note that these two conditions each cover a different kind of consideration. The first looks at the consequences of the action. Will this action harm others? The second looks at the intent of the action. What motivated the action? In general, considerations of morality tend to fall into these two buckets of consequences and intent.

Back to the task at hand. Given that this is a PSA question, it’s important to note which conclusions are reachable and which are unreachable.

A reachable conclusion is that the government is justified in interfering with the individual’s action. To reach this conclusion, we just have to show that (1) and (2) are satisfied.

An unreachable conclusion is that the government is not justified in interfering with the individual’s action. There’s simply nothing we can show to trigger the conditional in that way. If we wanted to reach the unjustified conclusion, we need to know what the necessary conditions of justified are, fail those conditions, then contrapose back.

This analysis is helpful in eliminating Answer Choice (A). It tries to conclude that the government is unjustified in interfering with Jerry’s moviemaking. We don’t need to read the rest of the argument. There’s nothing that the premise can state that will make use of the conditional to reach this conclusion. (A) is therefore wrong on its logic alone. In other words, it makes a sufficiency-necessity confusion, the oldest mistake in the book. We can stop here, but for review, look at the premise. It says that Jerry’s action (moviemaking) doesn’t harm and won’t increase the likelihood of harming anyone. It also says that it is motivated by a desire to help others. So it fails both (1) and (2). But failing sufficient conditions just makes the rule go away. It doesn’t trigger anything. Yet, (A) thinks it triggers the failure of the necessary condition. That’s textbook sufficiency-necessity confusion.

Contrast this with Correct Answer Choice (E). It concludes that the government is justified in preventing Jill from giving her speech. That’s a reachable conclusion. We just need to show that Jill’s speech satisfies (1) and (2). And it does. Her speech “would most likely have caused a riot and people would have gotten hurt.” That’s physical harm to others. And her speech was “to further her own political ambitions.” That’s a selfish motivation and hence not a motivation to help others.

Answer Choice (B) concludes that the government is justified in fining the neighbor for not mowing his lawn. That’s a reachable conclusion. We just need to show that the neighbor’s not mowing his lawn satisfies (1) and (2). (1) is problematic. It’s not pleasant to look at an unkempt lawn, but that’s not physical harm to others. We don’t need to consider (2) but probably the neighbor’s decision to not mow his lawn was selfishly motivated. He was probably just feeling lazy.

Answer Choice (C) concludes that the government is justified in requiring motorcyclists to wear helmets. That’s a reachable conclusion. We just need to show that motorcyclists’ not wearing helmets satisfies (1) and (2). Again, we have a problem for (1). It’s not clear that their failure to wear helmets would increase harm to others, whatever the consequences of harm are for themselves. We don’t need to consider (2) but probably their decision to not to wear helmets was selfishly motivated. They probably were feeling lazy, wanted to look cool, or have a death wish.

Answer Choice (D) concludes that the government is justified in suspending Z’s license to test new drugs. That’s a reachable conclusion. We just need to show that Z’s testing new drugs satisfies (1) and (2). Again, we have a problem for (1). It’s not clear that their testing of new drugs would increase harm to others. In fact, if they’re a drug company, then it’s more likely that their testing of new drugs would do just the opposite. It would help others alleviate pain and suffering. We don’t need to consider (2) but here the argument makes explicit that their motivation is selfish and not to help others.


13 comments

This is an Inference question.

The question stem says “properly inferred” from the sociologist's perspective. Inference from others' perspective is a question type that we see more often in RC.

The stimulus starts by telling us what rational choice theory says about what causes support for political parties. It says that popular support for political parties is caused by individual voters making deliberate decisions to support those parties whose policies they believe will economically benefit them. In other words, individuals' beliefs about the economic consequences of a particular party's policies cause those individuals to support those parties. This causal relationship is what is meant by “sufficiently explained.”

But the sociologists don't agree. They oppose rational choice theory on the premise that a complex phenomenon like the rise of a political organization or party cannot be caused by a simple phenomenon.

What is this “simple phenomenon”? It must be the individual voters making economic decisions to support political parties, which implies that it must not be a complex phenomenon. This is what Correct Answer Choice (A) says. Sociologists believe that economically motivated decisions by voters need not constitute a complex phenomenon. We are getting hints of an NA question. Note how (A) could have stated this much more strongly. Economically motivated decisions by voters constitute a simple phenomenon. That would have been correct as well. But the test writers took it one step further and stated an inference of that statement.

Answer Choice (B) says a complex phenomenon generally will have many complex causes. This is unsupported. The sociologists only said that a complex phenomenon cannot be caused by a simple phenomenon. This leaves open several possibilities. Perhaps they believe that a complex phenomenon can be caused by many simple phenomena. Or perhaps they believe that a complex phenomenon can be caused by a single complex phenomenon. We’d have to dismiss those alternatives without warrant in order to arrive at (B).

Answer Choice (C) says political phenomena often have religious and cultural causes as well as economic ones. This is even more unsupported. Note the same reasoning in (B) applies here. Additionally, (C) draws an inference to religious and cultural causes on the basis of nothing.

Answer Choice (D) says popular support for political parties is never a complex phenomenon. This is anti-supported. The sociologist called the rise of a political organization a complex phenomenon. Within the context of the stimulus, the rise of the political organization is synonymous with popular support for a political party.

Answer Choice (E) says the decisions of individual voters are not usually influenced by their beliefs about which policies will yield them the greatest economic advantage. This is unsupported. The stimulus talks about a narrow political relationship. It examines the causes of the rise of popular political parties. (E) talks about a much broader political relationship, the causes of individual voting decisions. The stimulus has very little to say about what generally causes (influences) or doesn't cause voters to cast their vote one way or another.


11 comments

This is an Inference question.

The question stem says “properly inferred.” This is a challenging question because there is so much information in the stimulus that is all connected. That can easily induce panic as you scramble to draw all the connections and valid inferences. Strategically, you shouldn't do that. The more connected information a stimulus contains, the more valid inferences there are to be drawn, the less you are able to anticipate the correct answer choice. In stimuli like those, POE is the better approach.

The first line in the stimulus about the study of primates being interesting is the only irrelevant fact. Everything else is fair game.

Only primates have opposable thumbs. Lemurs are lower primates (a subset of primates.) And lemurs are the only primates indigenous to Madagascar. Some species of lemurs are the only living lower primates that are diurnal. They go ahead and define diurnal for us but the answer choices never swapped out the term for its definition so we don't need to pay attention to it. Finally, all higher primates (a subset of primates) are thought to have evolved from a single diurnal species of lower primates.

Lots of information. Let’s POE.

The last piece of information is what sets up the trap in Answer Choice (A). It says that the chimpanzee, a higher primate, evolved from the lemur. This is not a proper inference. This is unsupported. All we can say is that the chimpanzee, being a higher primate, is thought to have evolved from a single diurnal species of lower primates. Which one, though? Must it be the lemur because the lemur is a diurnal species of lower primates and it is the only living one? No. Being alive isn’t required. We’re talking about evolution here. Most ancestor species are extinct. There may well have been other extinct, diurnal species of lower primates. One of those extinct species may well be the evolutionary starting point of all higher primates.

Correct Answer Choice (B) says no primates indigenous to Madagascar are diurnal higher primates. We can transform this into the following logically equivalent claim: all primates indigenous to Madagascar are not diurnal higher primates. This must be true. The stimulus says that the only primates indigenous to Madagascar are lemurs and that lemurs are all lower primates. It is implied that lower primates cannot be higher primates, diurnal or otherwise. This is the conditional chain: prim-indig-M → lemur → low-prim

Answer Choice (C) says no higher primate is nocturnal. This is unsupported. We simply have no idea if higher primates are nocturnal or diurnal or anything else. The only piece of information we have about higher primates is that they are thought to have evolved from a single diurnal species of lower primates.

Answer Choice (D) says there are some lemurs without opposable thumbs. This is unsupported. The stimulus says only primates have opposable thumbs. That means if something is not a primate then it doesn't have opposable thumbs. But lemurs are primates. Sufficient condition failed, rule goes away.

Answer Choice (E) says there are no nocturnal lemurs. This is unsupported. The stimulus tells us that some species of lemurs are diurnal. Maybe all species of lemurs are diurnal, maybe not.


10 comments

This is a Weaken question.

The author concludes that the government should require drug companies to notify consumers of all known drug-related interactions. Why? Because even relatively minor drug-related interactions can still be harmful to patients. That’s the major premise. Why should we believe this? The author provides an example: aspirin and fruit juice interact to render the aspirin ineffective. People who are unaware of this end up taking an incorrect dosage.

The logic of this argument is cost-benefit analysis. We consider a cost and from that consideration draw a conclusion about what should be done. As with all cost-benefit analysis questions, there are other factors to consider.

Correct Answer Choice (A) provides a counterbalancing consideration. It says that providing information on minor drug-related interactions would detract from a patient’s attention to serious interactions. That seems like a pretty compelling cost which weighs against the stated benefit in the stimulus. In fact, it seems like it outweighs the benefit, though that isn’t required in order to weaken the reasoning.

Answer Choice (B) says many drugs have fewer documented drug-related interactions than does aspirin. This is irrelevant. The argument used aspirin merely as an example. It never assumed that aspirin was the drug with the least documented interactions.

Answer Choice (C) says providing information about all drug-related interactions would result in only negligible price increases for consumers. This is a consideration on the benefit side of the scales. This wouldn’t weaken the argument.

Answer Choice (D) says current research is such that many drug-related interactions have not yet been identified. Of course not. Why should we expect we’d have completely identified all the interactions? But the mere fact that there are interactions that we don’t yet know about weighs on neither side of the scales. If the concern here is that there may be some as yet unidentified interaction X that’s really dangerous, what can we do? We certainly can’t print X on the label because we don’t know about X. That’s what it means to be unidentified.

Answer Choice (E) says pharmacists usually draw patients’ attention to printed warnings that are provided with drugs. This also wouldn’t sway the argument in either direction. Pharmacists are merely acting as a highlighter, reinforcing the already printed warnings. We’re talking about whether to add additional warnings onto the label.


8 comments

Newspaper columnist: What caused the current recession is a hotly debated question. It is a mistake, however, to assume that answering this question is essential to improving the economy. Corrective lenses, after all, were an effective treatment for myopia long before the cause was known to be genetic.

Summarize Argument
The author concludes that we do not need to figure out the cause of the current recession in order to improve the economy. This is supported by an analogy to corrective lenses. The author points out that corrective lenses were a solution for myopia even before we figured out the cause of myopia.

Identify Conclusion
The conclusion is the author’s assessment that identifying the cause of the current recession is not necessary to improve the economy: “It is a mistake, however, to assume that answering this question is essential to improving the economy.”

A
Solving a problem never requires finding the root cause of the problem.
This is too extreme. The author’s conclusion is not about problems generally. It’s about what’s necessary to improve the economy.
B
Knowing the cause of the current recession would not necessarily enable people to find a solution to it.
This twists the conclusion. The author believes knowing the cause is not essential for improving the economy. But that doesn’t mean the author believes it wouldn’t be sufficient for identifying a solution.
C
The question of what caused the current recession is subject to considerable debate.
This is context. The author’s conclusion concerns whether answering this question is essential to improving the economy.
D
One need not ascertain the cause of the current recession in order to improve the economy.
This is a paraphrase of the conclusion.
E
Long before the cause of myopia was known to be genetic, corrective lenses were being used as an effective treatment for the disease.
This is the analogy used as support for the conclusion.

5 comments

Radioactive elements may have been created when the universe began. However, even if this occurred, these elements are clearly still being created in the universe today. Radioactive elements are unstable, so most of them decay within at most a few million years into other, nonradioactive elements. So, if no new radioactive elements had been created after the universe began, almost no radioactive elements would be left in the universe today, but there is an abundance of such elements.

Summarize Argument
The author concludes that radioactive elements are still being created in the universe. This conclusion is based on the contrapositive of a conditional statement offered as support. If no new radioactive elements had been created after the universe began, then there would be almost no radioactive elements left in the universe today. But we know there’s a large amount of such elements today.

Identify Conclusion
The conclusion is the author’s assertion that radioactive elements are still being created today: “[T]hese elements are clearly still being created in the universe today.”

A
Any radioactive elements created when the universe began have probably decayed into other, nonradioactive elements.
This is an inference we can draw from the third sentence. Based on this, the author believes that if no radioactive elements had been created after the universe began, we wouldn’t find many radioactive elements today. This conditional is used to support the author’s conclusion.
B
Radioactive elements are being created in the universe today.
This is a paraphrase of the author’s conclusion.
C
If no new radioactive elements had been created after the universe began, almost no radioactive elements would be left in the universe today.
This is a subsidiary conclusion based on the premise that most radioactive elements decay within a few million years. The author uses this conditional to conclude that radioactive elements were created after the universe began.
D
It is possible that radioactive elements were created when the universe began.
This is context. The author’s argument concerns whether there were radioactive elements created after the universe began.
E
Due to their instability, most of the universe’s radioactive elements decay within at most a few million years into other, nonradioactive elements.
This is a premise.

2 comments

This is a Must Be True question.

This question tests your ability to manipulate grammar to reveal underlying logical relationships between sets. In particular, we are dealing with sets of intersections and supersets and subsets.

The stimulus states that if a belief is based on information from a reliable source, then it is reasonable to maintain that belief. This is a standard conditional claim using the “if... then...” formulation. Let's kick the idea of “a belief” up into the domain. This will simplify the analysis. You wouldn't know at this moment to do this. You have to finish reading the stimulus.

Kicking the idea of “a belief” as the subject up into the domain, we get to talk about the properties of beliefs. And it's those properties that have a sufficiency necessity relationship. The sufficient property is “based on reliable information.” If that's true, then the necessary property is “reasonableness.”

The next claim is an intersection claim. We know this from the presence of the word “some.” Some beliefs are based on information from a reliable source and yet are neither self-evident nor grounded in observable evidence. This is where you might notice that the subject once again is belief. The sentence here is talking about a triple intersection between three different sets of properties of beliefs.

  1. based on reliable information (same set as the sufficient condition in the previous sentence)
  2. not self-evident
  3. not grounded in observable evidence

These three sets have an intersection. But we also know from the first sentence that if based on reliable information, then reasonable. That implies “reasonable” gets to join this intersection with “not self-evident” and “not grounded in observable evidence” as well.

Using logic, first, represent the intersection of 2 and 3 as simply A. B will represent based on reliable information. C will represent reasonable. The triple intersection can now be represented as A ←s→ B. The conditional is B → C. The valid inference is A ←s→ C. Again, that’s just the triple intersection between “reasonable” and 2 and 3.

If that was confusing, consider this. If a cat is mild-mannered, then it’s domestic. Some mild-mannered cats are large and fluffy. Therefore, some domestic cats are large and fluffy. That’s analogous to the question here. Kick the subject “cats” up into the domain. The “some” premise describes a triple intersection between mild-mannered, large, and fluffy. Because mild-mannered implies domestic, we know that domestic also intersects with large and fluffy.

Hopefully that clears up the logic. Translating it back into English reveals that we have many options. “Some domestic cats are large and fluffy” is probably the most straightforward translation. But we could also say, “Among the large cats, some domestic ones are fluffy.” Or we could say, “Among the fluffy cats, some large ones are domestic.” The order of the modifiers in “some” intersections doesn’t matter. “Some” can be reversibly read.

So, there are many ways to translate the valid inference from the actual stimulus back into English. We could say, “Some beliefs that are neither self-evident nor grounded in observable evidence are nonetheless reasonable.” Or we could state this as Correct Answer Choice (E) does. Among reasonable beliefs that are not self-evident, there are some beliefs that are not grounded in observable evidence. Or even differently still. Grammar is the reason for this flexibility. As long as you know that you're just trying to express a triple intersection, you should be all set.

Answer Choice (A) says beliefs for which a person does not have observable evidence are unreasonable. This is a mishmash of the concepts in the stimulus. What must be true is that some beliefs for which a person does not have observable evidence are reasonable. Or alternatively, we could say that unreasonable beliefs must not be based on unreliable information.

Answer Choice (B) says beliefs based on information from a reliable source are self-evident. This is also a mishmash of the concepts above. We have no information to make claims about self-evident beliefs.

Answer Choice (C) says all reasonable beliefs for which a person has no observable evidence are based on information from a reliable source. This does not validly follow from the premises. But if we changed the quantifier “all” into the quantifier “some,” then it would follow validly.

Answer Choice (D) says if the belief is not grounded in observable evidence then it is not self-evident either. This also does not follow. The relationship between these two concepts is only one of intersection. We don't know if the two sets have a superset-subset relationship.


13 comments

This is a Weaken question.

The argument starts with a report of what some people believe. They believe that the economic benefits of genetically engineered foods may be offset by the hidden health risks. That's the context, that's other people's position. The author, who is a scientist, signals the transition to argument with the word “however.” She hits us with her conclusion that the risk, referring to the risk to human health, is minimal. Why should we believe this conclusion? The rest of the stimulus provides reasons.

The first premise is that in most cases of deliberate alteration of a plant's genetic structure, only a single gene out of 750,000 is changed. She then says that since the change is so slight, it cannot have effects significant enough to be worrisome.

There are many assumptions in this argument but perhaps the most crucial assumption is whether a single gene change can cause effects significant enough to be worrisome. If it's true that the alteration of a single gene cannot have effects significant enough to be worrisome, then her conclusion follows pretty strongly. It does seem like the risk, from plant foods anyway, is minimal. However, if it's not true, if even a single genetic alteration is capable of producing significant effects to be worrisome, then her argument is severely weakened.

This is what Correct Answer Choice (D) reveals. It says that there are plants that are known to be toxic to some animals and whose toxicity is known to be affected by the alteration of a single gene. (D) reveals that indeed it is possible for the alteration of a single gene to have effects significant enough to be worrisome. This suggests that the crucial assumption in the argument is false. Note that this answer does not overshoot the requirement of Weaken questions. If you object to this answer on the grounds that we don't know if these plants in (D) are analogously similar to the plants that we're talking about in the stimulus, then you are trying to do too much. You're trying to prove the opposite of the conclusion. In other words, you're trying to prove that there is a significant risk to human health from the alteration of a single gene in, say, the corn that we grow. Indeed, to do that, we do need to show that the alteration of a single gene in corn results in toxicity. But that's not what we're trying to do. We're just trying to weaken the argument. (D) easily crosses that threshold.

Granted we didn't talk about the other assumptions that the argument made. Another big one is that the premises only talk about altering a plant's genetic code. The conclusion is more broadly stating that the risk of foodstuffs in general is minimal. That includes meat and other animal products as well.

Answer Choice (A) says the genetically engineered plants that have been developed so far have few advantages over plants that are not genetically engineered. If the argument above worked on a cost-benefit logic, then (A) might be relevant. Even so, (A) only explicitly confirms that there are advantages, just not that many of them. But the affirmation of the advantages that do exist, few as they are, has no bearing on any of the assumptions made in the actual argument.

Answer Choice (B) says whatever health risks there are in foods from genetically altered plants may be somewhat reduced by other factors. And then it goes on to state what those factors are. This being true does not weaken the argument. Notice that (B) doesn't take a position on whether health risks exist. It just says that if those health risks exist, they may be reduced. So in the world in which health risks exist, (B) provides ways to mitigate that risk. But that's not what we're talking about. We're trying to figure out if we are in a world where the risk is significant or if we are in a world where the risk is minimal.

Answer Choice (C) says scientists have yet to determine for each characteristic of some plants and animals used for food the precise location of the genes that determine a characteristic. If we are able to parse this statement, I think we'll see that it has no bearing on the argument. What the statement is saying is that we don't have a complete mapping of the observable characteristics (phenotype) of plants and animals onto the genes that are responsible for those characteristics. So, for example, we might observe that a particular variety of corn happens to be highly resistant to heat, or that it may be very fragrant, or that it grows really well regardless of the time of year. Those are all observable characteristics (phenotypes) that are controlled by genes. (C) is saying that we just don't know which genes control those characteristics. That's totally fine. The argument doesn't require a complete mapping of genes to observable characteristics. It just requires the assumption that a small change in a single gene can't have significant negative effects that we should worry about.

Answer Choice (E) says the research has shown that those consumers who are most strongly opposed to genetically altered foods tend to be the most ill-informed on the issue. Again this has no bearing on the argument. The argument is about whether a single genetic alteration can produce effects that warrant concern. How consumers feel about this is utterly irrelevant. (E) tells us that the consumers who are most strongly opposed are most ill-informed and the consumers that are least opposed are best-informed. That makes sense but it has nothing to do with the argument.


12 comments

This is a Strengthen question.

The argument begins with a phenomenon that a certain ancient society burned large areas of land. Naturally, we wonder why they did this. The author presents other people's hypothesis: they burned large areas of land to prepare the ground for planting, which means that the ancient society was beginning the transition to agriculture.

To test this hypothesis, we can check its predictions. One prediction would be evidence of agriculture. If it's true that they burned the ground in preparation for planting, then we should expect to find evidence of agriculture. But we have little evidence of cultivation after the fires. This strongly implies that the other people's hypothesis of transition to agriculture is wrong. And so the author concludes it is likely that the society was still a hunter-gatherer society.

Now, one quick assumption you might've noticed is whether ancient societies fall into the binary buckets of either agricultural or hunter-gatherer. That is something to keep in mind, but as it turns out, those two buckets do largely capture all societies. The answer choices don't try to undercut that assumption.

But don't forget that we still have this phenomenon presented in the beginning argument. The author hasn't given an explanation of why the ancient society burned large areas of land. She has only, rather effectively, disposed of a bad explanation.

This is where Correct Answer Choice (D) improves the reasoning of the argument. It says hunter-gatherer societies are known to have used fire to move animal populations from one area to another. This presents a plausible explanation of the phenomenon unexplained in the original argument. If this is true, then that phenomenon itself becomes support for the author's conclusion that the society was still a hunter-gatherer society.

Answer Choice (A) says many ancient cultures had agriculture before they began using fire to clear large tracts of land. This means that fire clearing of land is not necessary for the transition to agriculture. That's good to know if you were curious about early human civilization. But this has nothing to do with the argument. The fact is the particular ancient society we’re talking about did clear large areas of land with fire. We’re trying to figure out what that means about the status of their civilizational development.

Answer Choice (B) says hunter-gatherer societies use fire for cooking and for heat during cold weather. This doesn't affect the argument at all. The argument told us that this particular society used fire to burn large areas of land and then we try to argue that this particular society was still a hunter-gatherer society. Information about hunter-gatherer societies using fire to do other things doesn't help the claim.

Answer Choice (C) says many plants and trees have inedible seeds that are contained in hard shells and are released only when subjected to the heat of a great fire. This is probably the most attractive wrong answer choice because it also looks like it's trying to provide an explanation for the phenomenon described above. It's trying to suggest that the reason why the ancient society burned large areas of land was to extract the seeds from the hard shells. There are at least two problems with (C), however. The first problem is that the seeds are inedible. That means you can't eat them. So what are you trying to do by extracting them? One plausible explanation is that you're trying to plant them. But that's not good for this argument, because that suggests that the culture might have been agrarian. The other problem is that this explanation doesn't fit very well with the phenomenon. Even if it's true that the seeds are released only when subjected to the heat of a great fire, it's not clear that the way to extract a seed is to burn down an entire tract of land. Why not collect all the shells and just burn them? Wouldn’t that be easier than setting a whole forest on fire? Notice (D) doesn't suffer from this problem. The hypothesis fits the facts. If you're trying to move entire populations of animals, then burning large areas of land makes sense. The solution is at the right scale for the problem.

Answer Choice (E) says few early societies were aware that burning organic material can help create nutrients for soil. This suggests the preclusion of a potential explanation. Before reading (E), one potential explanation for why the ancient society burned large areas of land was to fertilize the soil. After reading (E), it seems less likely that that's what our ancient society was attempting to do. What is the significance of this? I suppose it's less likely now that our ancient society was agrarian. But this was already established in the argument.


5 comments

This is a Weaken question.

The ethicist begins her argument with a general rule: hospital staff must not deceive patients about their medical treatment. Then she talks about a specific case. Dr. Faris told a patient that medication A would help him sleep even though medication A has no known sleep-inducing properties. The ethicist then concludes that Dr. Faris has violated the rule.

She does concede the point that after taking medication A, the patient's sleep did in fact improve. That's a concession point because it doesn't help her argument.

This argument should remind you of PSA questions because those are the questions where we most frequently encounter general rules applied to specific cases. And if you were to treat this as a PSA question, where would you supply the missing assumption bridge? It has to be a bridge linking Dr. Faris's actions to deceiving the patient. If Dr. Faris's actions constitute deception, then he did violate the rule. Otherwise, he didn't.

So do they constitute deception? That depends on whether he knew medication A would help the patient sleep. If he knew it would, then he wasn’t trying to deceive the patient, he was just trying to be a doctor. If he knew it wouldn't, then he was trying to deceive the patient. The ethicist assumes that he knew it wouldn’t because medication A has no known sleep-inducing properties.

But, as the ethicist concedes, medication A did in fact help the patient sleep. So was that just a coincidence or is there something else going on? Is it possible for a medication with no known sleep-inducing properties to nonetheless help a patient sleep?

Correct Answer Choice (C) tells us that indeed it is possible. It says that medication A is a pain reliever that can indirectly lead to sleep due to a reduction in the patient's discomfort. This is a crucial clarification of the causal relationship between medication A and sleep. It respects what the premises said about medication A, that it has no known sleep-inducing properties. That's true. It doesn't directly cause sleep. But it indirectly causes sleep by reducing pain, which reduces discomfort, which facilitates sleep. Given what (C) reveals about medication A, the ethicist's argument falls apart. It seems far less likely now that Dr. Faris deceived his patient with the claim that medication A would help him sleep. Rather, it seems far more plausible that Dr. Faris knew that reducing pain would facilitate sleep.

Given what you know about the placebo effect, can you imagine a different formulation of (C) such that it would still be correct? It could go something like this: sleep, like pain reduction, is highly susceptible to the placebo effect and nearly all patients who are told that they are receiving a substance that would help them sleep will experience improved sleep regardless of whether that substance itself induces sleep. This answer would be very similar to the actual answer (C) because a placebo effect is simply one kind of indirect causal chain. The administration of the placebo causes internal chemical reactions in the body that cause sleep.

Answer Choice (A) says Dr. Faris was aware that medication A had no known sleep-inducing properties. This would strengthen the argument. The reason is because of an implicit requirement of the rule. Deception is an intentional act. An intent to deceive requires factual knowledge about the medication’s lack of sleep-inducing properties. But as we already discussed above, (A) wouldn’t strengthen the argument by much since it’s still possible that Dr. Faris knew that medication A would only indirectly help the patient sleep. So while (A) might be a correct answer for a Necessary Assumption question, it wouldn't be the correct answer choice for a PSA question.

Answer Choice (B) says a committee at the hospital is currently considering revisions to the hospital's ethics code. Who cares? (B) may as well have told us that the committee is considering what they're going to order for lunch. The ethics code is what it is regardless of what the committee is considering. We just have to answer the question of whether Dr. Faris violated the code as it currently is. That answer does not depend on what the committee ultimately decides to do with the code.

Answer Choice (D) says several other members of the hospital staff prescribed medication A to patients who have trouble sleeping. This answer is reminiscent of many wrong answers in RRE questions. There, a recurring type of wrong answer is one that piles on additional phenomena in need of an explanation, rather than supplying the explanation. Do you see how this is similar? Our concern coming into the answers was whether Dr. Faris is in violation of the ethics code for having prescribed medication A. (D) doesn't answer that question. (D) just adds to the problem. Now I'm wondering whether these other members of the hospital staff are also in violation of the ethics code.

I suspect (D) might be baiting us to assume that if a sufficient number of people do a thing, then that thing isn’t wrong. But that’s a terrible assumption.

Answer Choice (E) says Dr. Faris knew that the patient was not taking any other medication that has sleep-inducing properties. This is irrelevant because the argument never assumed anything about Dr. Faris's knowledge of whether the patient was already taking, say, melatonin or Ambien. The argument does assume that Dr. Faris knew that medication A has no known sleep-inducing properties, as we discussed in (A), and that Dr. Faris knew that medication A also wouldn't indirectly facilitate sleep. If those two assumptions can be established, then it does look like Dr. Faris has violated this specific ethics code.


Comment on this